« first day (2460 days earlier)      last day (2565 days later) » 

6:02 PM
Yeah, it seems Mathematica can only do multivariable limits along lines.
It's not a theoretical mathematician.
 
aha
 
tsk tsk tsk
 
should I use Taylor expansion to show it's not continuous? I get this:
$$
\frac{x^2y^2-x^2O(y^6)}{x^2+y^4},
$$
so I'm guessing I should show that $\begin{aligned}\frac{x^2y^2}{x^2+y^4} \end{aligned}$ is not continuous at $\vec 0$. I can't use homogeneity. Can I use the fact that:
$$
\left\vert\frac{x^2y^2}{x^2+y^4} \right\vert\geq \left\vert\frac{x^2y^2}{x^4+y^4} \right\vert,
$$
for $\vert x\vert,\vert y\vert<1$. Because we know that the limit of $\begin{aligned}\left\vert\frac{x^2y^2}{x^2+y^4} \right\vert
 
Oh, maybe I'm wrong on the continuity. So, yeah, substitute $u=y^2$.
I was wrong.
My apologies.
 
oh yea, I see
You remove $y^4$ from the demoninator, and then you get $\vert y^2\vert$, which goes to zero
 
6:09 PM
Huh?
Oh.
I wouldn't do that.
But OK.
For differentiability, you probably want to leave the $y^4=u^2$ in the denominator.
 
$$
\left\vert\frac{x^2y^2}{x^2+y^4} \right\vert\leq\left\vert\frac{x^2y^2}{x^2} \right\vert=y^2\to 0
$$
I also remove $y^4$ to show it's differentiable
I'll show you
 
Yeah, OK.
I get it.
 
okay
lol, so no mathematica?
:P
 
Right.
 
too bad
 
6:14 PM
What would you do with $\dfrac{x\sin(y^2)}{x^2+y^4}$?
 
Should I first show if it's continuous?
 
right
 
That boils down to showing that $\begin{aligned}\frac{xy^2}{x^2+y^4} \end{aligned}$ is continuous
 
for $u=y^2$, we get that it's homogeneous < 0
so I see that that's not continuous, but I would have to think about this substitution
 
6:17 PM
OK. Cool.
 
I would use this then: $f(tx,\sqrt{ty})$?
and then show that $f(tx,\sqrt{ty})=g(tx,ty)$, where $g$ is homogeneous with $\alpha<0$
 
No.
For one thing, $\sqrt y$ isn't good.
But how do you go from substituting $u=y^2$ to taking $\sqrt y$?
Also you keep saying homogeneous of degree <0. Isn't it =0?
 
oh yea, you're right :P
$$
f(tx,\sqrt{ty})=\frac{t^2xy}{t^2(x^2+y^2)}:=g(tx,ty)
$$
so for $t,y>0$, we could show it
 
Oh, I see what you were thinking. I'm thinking $g(x,u) = f(x,\sqrt{|u|})$.
 
Anonymous
Does $$y=(x-1)(x-2)^2(x-3)^4(x-4)^3$$ have an inflection point at $x=3$ or not? I think it should have since it has even number of roots $x=3$ (i.e. 4) [Can we say it without calculating the second derivative explicitly?] Ideas anyone?
 
6:23 PM
But you'd better put in absolute value in yours.
 
ah yea, that's better!
 
No, @blue. Does it change concavity/convexity there?
@Alessandro
 
Anonymous
Oh wait. I'm wrong
 
Yep.
:)
 
LOL I just spent 15 minutes talking to a guy on PSE
 
Anonymous
6:25 PM
x=4 is an inflection I think
 
It looks like some positive constant times $-(x-3)^4$ near $x=3$, @blue.
 
ya, @blue
 
only to discover we study in the same building
 
LOL, @Danu.
 
$y = x^{2n+1}$ have inflection points at $0$, and locally think about it like that. That's all you need to understand inflection points.
 
Anonymous
6:26 PM
I get the logic now. For even no. of roots the concavity does not change.
 
There's actually another occasional chatter here who's studying math in Munich.
Just think about the graph, @blue.
 
@TedShifrin what?
 
@TedShifrin Really? Who?
Maybe I know him/her
 
Hey @Alessandro!
 
I only know 'cuz he emailed me. I don't know if i should say.
 
6:27 PM
Hi @Dami
 
I might let him know I'm going to be there, though.
 
@TedShifrin Just the MSE handle?
 
I hesitate to make things public that were told privately.
 
If he does any kind of geometry/topology stuff, I probably know him in real life!
 
@Ted Is that our yellow friend?
 
Anonymous
6:28 PM
@BalarkaSen Ah, the wavy curve method helps
 
Just starting undergraduate stuff.
 
Anonymous
Got it
 
@TedShifrin Oh, okay.
 
Why did you ping me Ted?
 
Then there's no point in me asking :P Tell him to take Leeb's courses.
 
6:28 PM
@blue Didn't know that had a name.
 
And Kotschick's.
 
It appeared you had just entered, @Alessandro. Saying hi.
 
(and Hamilton if they're looking for precise arguments, which may or may not be very tedious)
 
@Danu: He's just learning basic analysis/algebra/linear algebra.
 
Then probably not who I am thinking of.
 
6:29 PM
I have no idea whom you're thinking of, @Balarka.
 
@TedShifrin I think Leeb teaches some undergrad stuff most semesters. This semester the topology of surfaces that I'm grading for, but sometimes also easier stuff.
 
bananas. I forget where he studies.
 
Oh yeah, how's that undergraduate geometry going, @Danu?
 
Ah, I see, weird, I've been there for a while!
 
They should really try to avoid Zenk though. His love of terrible notation and general tediousness really turns people off of math.
 
6:30 PM
oh, bananas. He's elsewhere.
Apologies, Alessandro.
 
@TedShifrin The first sheet is out, but I won't grade that one because I'm going to that conference tomorrow.
 
I should really stop listening to music which breaks my ears. They're so damn good tho.
 
The first sheet asked them to prove (in steps) what the motions of $\Bbb R^n$ are @Ted
 
No problem Ted, I just thought I missed some messages I should have read :P
 
That's a good exercise, @Danu. My students would in general not have been strong enough for that. It's actually done in the appendix in my notes.
 
6:33 PM
In the first sheet, too. I was pretty impressed.
 
Isometries of R^n?
 
@Alessandro Have you seen the Vitali covering theorem?
 
Leeb does like to make hard exercises... I guess 75% of them will despair (and I guess my grading friends will tell me if they did :P)
 
right, @Balarka ... I remember as an undergrad being shocked that they all had to be linear :P
 
6:34 PM
Oh we had that as a challenge problem in Sally's book
 
I forget how to do that. Can I overkill and use that it must preserve geodesics? If every straightline is sent to straightlines I think you should be able to see it's linear.
 
hehe
 
I think that's cheating, Balarka.
 
OVERKILL indeed
 
:D
well, a proof anyhow!
 
6:35 PM
FATALITY
 
Go on and do it, Balarka.
 
Is that overkill? In spivak's book on diff geo that's how it's proved in the exercises there i think
 
And @Alessandro so a Vitali covering of a (let's say bounded) set $A\subset \mathbb{R}^n$ is a cover with balls such that for any $x\in A$ and any $\epsilon$, there's a ball of radius less than $\epsilon$ containing $x$
 
isometries preserve distance
 
Ok, so how does one really do it? Without cheating?
 
6:36 PM
So the way they are supposed to do it
 
One first proves (fixing origin) that it's linear.
 
prove they are collinearities. ;-)
 
is first prove that the fixed point set is affine (or empty)
 
prove a distance preserving map is a collinearity.
 
I don't even need to do that, @Danu. By translating, I can assume $f(0)=0$.
 
6:37 PM
that might be sufficient.
 
I see, @Ted.
 
@TedShifrin I'm just relaying the formulation of the exercise.
 
I know :P
 
Ok, that's easy enough.
 
Is it, @Balarka?
I thought you meant proving linear.
 
6:38 PM
then what you said about proving linear @Ted and then the statement
 
And the theorem is that given a Vitali covering of a set $A$, you can find countably many disjoint sets which cover almost all of it @Alessandro
 
weird, @Danu ...
 
I see @dami
 
@TedShifrin Why?
 
Sounds reasonable, unlike many things in measure theory
 
6:40 PM
I don't see why I have to mess with proving there's a fixed point to start with. That isn't obvious to me.
 
doesn't really matter
 
@TedShifrin now I know we were discussing what the avatar of yours is in terms of geometry, but what software did you use to make it? I've seen quite a few models with that same sort of look on here and I'm curious what makes them. :-)
is it CAD?
 
I did it in Mathematica centuries ago.
 
ah
that explains it
 
The argument (in brief) is that you can take those sets of radius larger than, say, 1, and throw them out, since every point is covered by an arbitrarily small ball. Now, you take a ball with radius greater than 0.9 (or any constant less than 1) times the sup of the radius of the balls, then take another at least 0.9 times the sup of everything disjoint from the first, and keep going
 
6:42 PM
The professor I last semester mentioned people do a lot of 3D stuff in it with math.
makes sense though given it's a math software. :p
 
I have no experience with it, GreatDuck
Oh, you mean he was referring to Mathematica? Yeah, sure.
 
@TedShifrin Sure. If $f$ fixes $0$, I think I can even show that it preserves the dot product from the norm-preserving condition.
 
OK.
Then why linear? :)
 
Hmm.
 
6:46 PM
So you get a sequence of balls $B_1,B_2,\ldots$, with radii tending to $0$ (since $A$ is bounded). Given some $x\in A\setminus \bigcup_i B_i$, you choose some ball $B$ in the covering containing it. We know that there is some $B_k$ that intersects $B$, since otherwise we would have chosen it instead of the way smaller ones down the chain. Take the first one that does it, $B_k$, and note that you can scale $B_k$ to contain $B$, say by a constant $c$.
 
I wanted to look at the orthogonal matrix which does the same thing it does to the standard basis vectors.
But I was wrong.
 
Oh?
 
Ah, wait, I think isometries agreeing on the standard basis vectors are the same. This should be provable.
 
Then $cB_1,cB_2,\ldots$ will necessarily cover all of $A$, so that $\sum_i \mu(B_i) \ge \frac{1}{c^n}\mu(A)$ (where $n$ is the dimension), so that there exists an $N$ such that $\mu(B_1) + \ldots + \mu(B_N) \ge \frac{1}{2c^n}\mu(A)$.
 
why not just show that it has to be given by that linear map directly?
 
6:48 PM
How would I do that, though?
 
You've thought about it long and hard?
 
Actually wait I forgot one step in the algorithm which was to throw out the balls that were disjoint from $A$. But yeah, so we then have that finitely many of the balls will cover some fixed portion of $A$ (since $c$ only depends on the constant we choose, in this case 0.9), so then you repeat the process for $A\setminus (B_1 \cup \ldots \cup B_N)$ and continue, covering almost all of it with countably many sets
@Alessandro
 
I'll read that later, thanks! I have to run away now
 
Aight, see you!
 
Re: limits and mathematica, you can do 1D right- vs. left- limits by an appropriate Assumptions call. @TedShifrin
But that's just 1D.
 
7:00 PM
The issue was 2D.
 
Yeah.
 
@Ted Oh, I see. $f(x) \cdot f(\mathbf{e}_i) = x \cdot e_i$. Since $f(\mathbf{e}_i) = A \mathbf{e}_i$, $x \cdot e_i = f(x) \cdot A \mathbf{e}_i = A^T f(x) \cdot \mathbf{e_i}$. $A^T = A^{-1}$ (follows from orthonormality), so you have $A^{-1} \circ f$ is identity.
That took longer than it should have.
 
OK.
 
@Ted I think it should be done by the geodesic preservation though. That not only says straightlines goes to straightlines, but that your map is affine on the straightlines.
 
well, go ahead and do it.
 
7:06 PM
*doable, not done.
 
Back later.
 
See ya.
 
I should head out and actually work on my essay, see you guys around!
 
@TedShifrin looking around at other concepts that might be relevant to a problem I'm trying to solve. Has anyone in the past used other sorts of differential algebrae to solve differential equations through some connection between their solution sets?
just curious
 
How can I show that $f$ is differentiable at $\vec 0$, where
$$
f(x,y)=x^2\log(x^4+y^2),
$$
and $f(0,0)=0$. I’ve already shown that $f$ is continuous at $\vec 0$. I started off by calculation the first partial derivative:
$$
D_1f(\vec 0)=\lim_{t\to 0}t\log t^4,
$$
however, I don’t know how to calculate this limit even. I looked at the plot, and it seems that $D_f(\vec 0)=D_2f(\vec 0)=0$, so apparently $t$ goes faster to zero then $\log t^4$ goes to minus infinity. How can I show this? Can I use Taylor? Should I evaluate then at $x=1$? This would yield:
 
7:19 PM
@ShaVuklia confirm differentiability by showing the partial derivatives exist methinks.
plug in the values at 0,0
and see if any vector dot product shows undefined values
 
@TheGreatDuck well i'm struggling with showing the first partial derivative exists
 
why are you using limits?
just take the derivative with respect to x?
 
can't do that, because it's defined piecewise
 
LOL!
if you've found continuity at 0 then that f(x,y) = __ thing is the only thing you need!
i assumed you were saying the value at 0 when evaluated
just take the partial derivatives of the main bulk. That should work for ya
see if it matches up with what you'd expect from the graph of the function
 
7:27 PM
really?
that doesn't make sense to me :P
because say a function is defined piecewise, and it's continuous on it's domain
then you're not free to show which "bulk" you take for your partial derivatives
 
Typically one has to do casework in such instances.
 
@ShaVuklia if a function is continuous, then its limits equal the point
the piecewise defined is just a single point
 
I'm never confident when it comes to multivariable limits these days, so I'm not going to weigh in.
 
@Semiclassical limits in general
 
No problem @Semi I've asked on the main site!
 
7:31 PM
Neat.
 
@ShaVuklia sure that f(0,0) = 0 isn't just telling you the value when plugging 0,0 into the other expression...
cause I think that is the value.
 
@ShaVukila You want to show $\lim_{t \to 0^+} t\log(t) = 0$, right? Note that $\lim_{t \to 0} t^t = 1$, and $t \log(t) = \log(t^t)$. You can push the limit inside the log (why?)
 
@BalarkaSen nooo....
 
let me think, @balarka!
 
@TheGreatDuck Huh?
 
7:32 PM
she wants to prove that the function is differentiable at 0!
 
A bit of a self-imposed challenge: express the number 13256278887989457651018865901401704640 in 40 characters or less using standard mathematical terminology (+, *, ^, etc.)
 
Look at the question.
She's computing $D_1f(0)$
 
derivative is continuous by definition...
 
??? Derivative is certainly not continuous by definition
 
it is either continuous or undefined.
 
7:34 PM
lolno
absolutely not
 
Garbage talk.
 
it cannot be discontinuous at a defined value
 
Again garbage.
 
really, what is the derivative at a sharp corner?
 
55
Q: Discontinuous derivative.

user58273Could someone give an example of a ‘very’ discontinuous derivative? I myself can only come up with examples where the derivative is discontinuous at only one point. I am assuming the function is real-valued and defined on a bounded interval.

check here for example
 
7:34 PM
I think he means the function is continuous, not the derivative itself
 
no
im saying the derivative cannot jump
 
A derivative can be awfully discontinuous
 
The derivative can jump.
 
It's true that $|x|$ does not give a counterexample.
But that's not the example they have in mind.
 
@Sha Nah, he's saying nonsense.
 
7:35 PM
@BalarkaSen at the point it jumps, it is undefined.
 
All wrong.
 
Have you looked at the example?
 
yeah, that is oscillation
not jumping
 
it's still discontinuous
discontinuous isn't defined as "jumping"
 
7:36 PM
Anyhow, @Balarka, I've never pushed the limit inside the log
 
that's the high-school handwavy explanation to avoid a proper definition
 
is that something I could show easily?
 
I specifically said I was referring to jump discontinuity
 
That is irrelevant to the question.
 
earlier before you came into the discussion
 
7:37 PM
3 mins ago, by TheGreatDuck
derivative is continuous by definition...
 
@Semiclassical oscillatory discontinuity is a bit of a grey area for some people and this function shouldn't have any oscillation so we can rule that out. That generally occurs with sine or modulo or something periodic. I assumed we were dealing with discontinuity of the second kind.
 
There is no grey area for math. Please don't answer people with handwavish things.
That confuses the OP even more and does little to no help
@ShaVukila Yep; the point is $\log$ is continuous at $0$, as a function on $\Bbb R^+$.
 
@BalarkaSen It's not as clear what counts as that type of discontinuity, and the discussion of it is irrelevant here anyways. We're trying to deal with the other kinds. That doesn't even have any relevance here.
@BalarkaSen besides, I was here first helping the person and you kind of jumped in.
 
I have no idea what you mean. In any case I don't care anymore, and going to ignore further handwaving garbage.
No you were not helping.
You thought you were but you were not
 
@BalarkaSen really? I was telling the person that once they proves continuity of the original function, they could ignore that piecewise defined issue they were saying stopped them from just taking the derivatives...
 
7:42 PM
@ShaVuklia
In case it's of any use
 
I'm pretty confused why the exercise asks them to prove thing about the fixed point set now too
 
@Danu wat
 
@SteamyRoot haha, unfortunately not, because my problem is calculating the first partial derivative :P
 
I'm referring to something from a while back.
 
how hard can it be to take a partial derivative? :p
 
7:46 PM
The hard part is (rigourously, not handwavey) proving that the partial derivative(s) are continous in $(0,0)$.
 
Defaulting back to the limit definition of the first partial if you're not sure strikes me as a good practice.
 
isn't the function the person gave smooth?
though whether or not that can have oscillation is beyond me
 
if it's piecewise, probably not.
 
You have to prove that even if it is.
 
@Semiclassical they claimed that they proved the limit at 0 is 0
 
7:48 PM
@SteamyRoot yea I got that as an answer too (I posted it on the main site 20 mins ago)
I wasn't aware of that theorem
but I'll look it up!
(if you can even call it a theorem :P)
 
which considering the piecewise defined point is just 0, I think that at least eliminates the piecewise definedness
 
That's a shaky argument.
 
really?
 
Either you have to include an additional argument as to why this function is well-behaved at zero (i.e. rigorously exclude the possibility of oscillations) or you just proceed directly from the limit definition.
 
if I have one point as a separate case, but then prove the outer function is continuous and has a limit approaching said value... is there any need to have it be piecewise defined anymore?
 
7:50 PM
That limit would only prove continuity. Also, 2D-limits are extremely tricky in general.
 
oh duh. XD
op claimed they did it though... could we progress from that?
 
As an example, I'm pretty sure that the $x^2\sin(1/x)$ example is 0 at 0 but doesn't have a continuous derivative.
So you'd need to rule out in a rigorous way that the function of interest can't do that.
...in which case you might as well just save the hassle and compute the limit definition of derivative directly.
But this is why I say that I tread lightly around 2D limits.
 
BUT the asker already computed the limit of the pre-differentiated function. So we're assuming continuity from the get go!
 
@ShaVuklia I think you're misunderstanding the answer provided
 
of course, the derivative might act weird
"I’ve already shown that f is continuous at 0⃗ ."
 
7:54 PM
And, again, the $x^2\sin(1/x)$ example is also "continuous at zero."
 
but that's not the function we were given
 
you mean this, @Steamy ?
 
Yeah
 
And? The point is that if you want to prove continuity of the derivative, it's not enough to say "oh, it doesn't do anything as weird as oscillate."
 
7:55 PM
ohh
 
You have to actually find some argument that rules out said behavior.
 
he meant that I check that the partial derivaties are continuous? @SteamyRoot
 
Yes, indeed.
 
which is weird, because I already wrote that I had trouble calculating the partial derivative in the first place
 
Finding the partial derivatives when $x \neq 0$ and $y \neq 0$ is easy
 
7:56 PM
yea
and for $x,y=0$ we resort to the definition
 
@Semiclassical except that we know that the multiplication of continuous functions is a continuous function and yada yada yada. Couldn't one just use that as an argument?
 
When $x = 0$ and $y \neq 0$, you can just use a single limit to prove continuity
same for $x \neq 0$ and $y = 0$
 
Maybe you can. I haven't paid enough attention to the example to say one way or the other.
 
yea, but that was exactly my initial problem, but I solved it using l'Hospital
which I didn't want to initially, because we haven't had l'Hospital "officially" yet, but whatever I guess
it's too easy to use it here
 
You only calculated the limit on a single line, though
Since you took $y = 0$ everywhere in your calculation
 
7:59 PM
@Semiclassical and I was which is why I was saying the continuity thing as I was presuming that the asker would understand i was referring to jump discontinuity which was their primary concern (since that is the potential discontinuity in the original function). It already went without saying that there was no oscillation. A simple graph can show that one is definitely not occurring.
 
Graphs are not a proof.
 
i never said it was
the asker is asking whether it is continuous or not. We don't have to be 100% rigorous in all of it. We're just conveying to them why it works. And intuition of that it most likely doesn't occur leads to a simple proof: just look at the fact that the partial derivatives are compositions of continuous functions in that neighborhood.
 
@Steamy I don't see why it's on a single line though, because we let $t\to 0$ "arbitrarily"?
I just had to calculate $D_1f(\vec0)$, right?
did I do that wrong?
you know what, I think it's solved. I get the idea:P I won't bother you guys with it anymore :P
 
you're not bothering us. I was just pointing out that maybe we're all overthinking it? :p
most problems in calculus classes tend to be simple and procedural if only to hammer the concepts into ones head through repetition. Of course, that can vary depending on the instructor.
 
Wait, how did you get that limit in the first place?
 
8:05 PM
@TedShifrin nice example of Russians leaving out "some details" (Vinberg's undergrad algebra book):
 
Shouldn't you get
$$\lim_{(x,y) \to (0,0)} 2x\log(x^4 + y^2) + \frac{4x^5}{x^4+y^2}$$
 
Footnote: Vinberg's book is officially grad level but it reads like undergrad
 
@SteamyRoot which partial derivative is that?
 
@Steamy yea that right
I'm struggling now showing $\lim_{(x,y)\to(0,0)}x\log(x^4+y^2)=0$. I can't use l'Hospital here right? Because we're dealing with two variables.
I also can't say $\vert\log(x^4+y^2)\vert\leq\vert\log(x^4)\vert$, because $\log$ is negative in our neighbourhood about zero
 
limits in 2D are evil
 
8:27 PM
@Danu I think that's a fair proof, though unless I'm mistaken the scalar multiplication claim is a little tricky
I guess that straight lines are geodesics is necessary.
 
Mike, do you see any way to use the fact that the fixed points of motions are affine subspaces to prove their general form?
The exercise my undergrads have to do makes them prove that first so I'd assume it is used but I dont' see a way to use it
(since there are motions whose fixed point sets are just one point)
 
Ah, actually that's crucial to my argument. It says that there's at least one fixed point which is essential
(It lets you define the zero vector!)
My geometric notion of addition involves the base point
 
8:48 PM

 Reading Group: A Survey of Measured G

This is a room for the reading group of the paper "A Survey of...
If anyone is interested in learning about measured group theory, we're starting a reading group in that chat room.
 
Is it hard to show the product rule for $f,g\colon\mathbb R^n\to\mathbb R^m$?
I'm surprised I can't find it anywhere online
It's easy to show $(f+g)'(\vec a)=f'(\vec a)+g'(\vec a)$ from the definition of differentiability
however, I know how to use the one-variable analogue in this case
maybe i could multiply some things in the definition
 
@ShaVuklia by product rule you mean $dg\circ f$ ?
 
@LeGrandDODOM I was about to type out the question elaborately on the main site:
this is how far I got. I'm going to try to multiply some stuff
and I'm not sure if that's what I mean ($dg\circ f$)
you decide, I guess :P
 
@ShaVuklia you mean $m=1$ ?
 
nope
orr
oh yea
that's right
the exercise says indeed $m=1$, but I thought they were referring to another variable $\mu$
it already made no sense :P
 

« first day (2460 days earlier)      last day (2565 days later) »